Difference between revisions of "2017 AMC 8 Problems/Problem 4"
Nukelauncher (talk | contribs) m (→Solution:) |
Nukelauncher (talk | contribs) |
||
Line 6: | Line 6: | ||
==Solution== | ==Solution== | ||
We can approximate <math>7,928,564</math> to <math>8,000,000,</math> and <math>0.000315</math> to <math>0.0003.</math> Multiplying the two yields <math>2400.</math> This gives our answer to be <math>\boxed{\textbf{(D)}\ 2400}.</math> | We can approximate <math>7,928,564</math> to <math>8,000,000,</math> and <math>0.000315</math> to <math>0.0003.</math> Multiplying the two yields <math>2400.</math> This gives our answer to be <math>\boxed{\textbf{(D)}\ 2400}.</math> | ||
+ | |||
+ | ==See Also== | ||
+ | {{AMC8 box|year=2017|num-b=3|num-a=5}} | ||
+ | |||
+ | {{MAA Notice}} |
Revision as of 14:24, 22 November 2017
Problem 4
When is multiplied by the product is closest to which of the following?
Solution
We can approximate to and to Multiplying the two yields This gives our answer to be
See Also
2017 AMC 8 (Problems • Answer Key • Resources) | ||
Preceded by Problem 3 |
Followed by Problem 5 | |
1 • 2 • 3 • 4 • 5 • 6 • 7 • 8 • 9 • 10 • 11 • 12 • 13 • 14 • 15 • 16 • 17 • 18 • 19 • 20 • 21 • 22 • 23 • 24 • 25 | ||
All AJHSME/AMC 8 Problems and Solutions |
The problems on this page are copyrighted by the Mathematical Association of America's American Mathematics Competitions.